in how many ways can alice give 12 apples to 3 children

Answers

Answer 1

Answer:

Step-by-step explanation:

give four apples to each child because 12/3 is 4

Answer 2

Answer:

91

Step-by-step explanation:


Related Questions

[tex]3-\sqrt{x} 1-16x^{2}[/tex]

Answers

Answer:

Step-by-step explanation:

This equation turns out to be a quartic. I'm not sure what should be done with. I can't believe you were asked to find its roots which are unbelievably complex. Here is a graph with the only 2 points that are easily found. If I am not solving what you need, please leave a note.

Triangle XYZ is isosceles. The measure of the vertex angle, Y, is twice the measure of a base angle. What is true about triangle XYZ? Select three options.

Answers

Answer:

A. Angle Y is a right angle.

B. The measure of angle Z is 45°.

E. The perpendicular bisector of creates two smaller isosceles triangles.

Step-by-step explanation:

Let x represent the measures of base angles X and Z. 2x is the measure of vertex angle Y.

x + x + 2x = 180°

x = 45°

2x = m∠Y = 90°

The triangle is an isosceles right triangle which has base angles of 45°.

The perpendicular bisector of line XZ creates two smaller isosceles triangles with acute angles of 45°

Answer:

The answers are A B E

Step-by-step explanation:

(06.01)

Write the following expression in exponential form:

1.6 × 1.6 × 1.6 × 1.6

41.6
1.64
1.6 × 4
1.6 + 4

Answers

Answer:

[tex]1.6^{4}[/tex]

Step-by-step explanation:

1.6 is multiplied by itself 4 times. This is represented in exponential form as

[tex]1.6^{4}[/tex]

The graph of a line is shown below. What is the equation of the line, in slope-intercept form, that is parallel to this line and has a y-intercept of 1?

Answers

Answer:

[tex]y = - \frac{3}{2} x + 1[/tex]

Step-by-step explanation:

Slope -intercept form: y= mx +c, where m is the slope and c is the y-intercept.

Parallel lines have the same slope. Let's find the slope of the given line.

Given points: (-2, 0) and (0, -3)

[tex]\boxed{slope = \frac{y1 - y2}{x1 - x2} }[/tex]

slope of given line

[tex] = \frac{0 - ( - 3)}{ - 2 - 0} [/tex]

[tex] = \frac{0 + 3}{ - 2} [/tex]

[tex] = - \frac{3}{2} [/tex]

[tex]y = - \frac{3}{2} x + c[/tex]

Given that the y- intercept is 1, c= 1.

[tex]y = - \frac{3}{2} x + 1[/tex]

what is the answer? I need help!! please and thank you

Answers

Answer:

B

Step-by-step explanation:

27%=0.27 and sqrt(2)<2.75

If Tan A=5/12 then find cot A, cos A and Sin A

Answers

Cot A=1/tan A=12/5

cos A= 12/13

sin A=5/13

Draw a right angled triangle

the hypotenuse is the longest side which is 13 using Pythagoras theorem

the side opposite the angle A is 5

the side closest to the angle A which is called the adjacent is 12

sinA =opp/hyp

cos A= adj/hyp

cotA =1/tanA=cos A/sinA

Note: Pythagoras theorem is

hyp²=opp²+adj²

Answer:

Step-by-step explanation:

[tex]tan \ A = \frac{5}{12}=\frac{opposite \ site}{adjacent \ side}[/tex]

hypotenuse² = (opposite side)² + (adjacent side)²

                      =  5² + 12²

                      = 25 + 144

                      = 169

hypotenuse = √169 = √13*13 = 13

[tex]Cot \ A = \frac{adjacent \ side}{opposite \ side}=\frac{12}{5}\\\\Cos \ A = \frac{adjacent \ side}{hypotenuse}=\frac{12}{13}\\\\Sin \ A = \frac{opposite \ side}{hypotenuse}=\frac{5}{13}[/tex]

help! please!!!!!! look at photo :))

Answers

Hey there!

We know that Danielle earns $10 per hour, so muliply that by 3 and get 30.

Because Danielle works an extra half an hour, divide 10 by 2 and get 5.

Danielle earns $35 in 3 hours and a half.

Hope this helps! Please mark me as brainliest!

Have a wonderful day :)


Charlene is a salesperson. Let y represent her total pay (in dollars). Let x represent the number of
items she sells. Suppose that x and y are related by the equation y=32x + 1900.
What is Charlene's total pay if she doesn't sell any items?
A. $32
B. $1,900
C. $3,200
D. $19

Answers

1,900
Multiply 32 by 0, and you get y=1,900

Which functions have a maximum value greater than the maximum of the function g(x) = -(x + 3)2 - 4?

Answers

Answer:

max: -4

Step-by-step explanation:

(x+3)^2 》0 mọi x

<=> -(x+3)^2 《0

<=> -(x+3)^2 -4 《 -4

Convert 75 mg into gram

Answers

75 mg into grams is 0.075

Answer:

[tex]{ \tt{1 \: mg = 1 \times {10}^{ - 3} \: g}} \\ { \tt{75 \: mg = (75 \times 1 \times {10}^{ - 3} ) \: g}} \\ { \bf{ = 75 \times 10 {}^{ - 3} \: grams}} \\ { \bf{ = 0.075 \: grams}}[/tex]

The angle made by the ladder with the ground is degrees, and the length of the ladder is inches.

Answers

Answer:

59.04°

58.31 inches

Step-by-step explanation:

The solution triangle is attached below :

Since we have a right angled triangle, we can apply trigonometry to obtain the angle ladder makes with the ground;

Let the angle = θ

Tanθ = opposite / Adjacent

Tanθ = 50/30

θ = tan^-1(50/30)

θ = 59.036°

θ = 59.04°

The length of ladder can be obtained using Pythagoras :

Length of ladder is the hypotenus :

Hence,

Hypotenus = √(adjacent² + opposite²)

Hypotenus = √(50² + 30²)

Hypotenus = √(2500 + 900)

Hypotenus = 58.309

Length of ladder = 58.31 inches

Answer:

59°

58.3 inches

Step-by-step explanation:

Here is the full question :

A ladder is placed 30 inches from a wall. It touches the wall at a height of 50 inches from the ground. The angle made by the ladder with the ground is degrees, and the length of the ladder is inches.

Please check the attached image for a diagram explaining this question

The angle the ladder makes with the ground is labelled x in the diagram

To find the value of x given the opposite and adjacent lengths, use tan

tan⁻¹ (opposite / adjacent)

tan⁻¹  (50 / 30)

tan⁻¹ 1.667

= 59°

the length of the ladder can be determined using Pythagoras theorem

The Pythagoras theorem : a² + b² = c²

where a = length

b = base

c =  hypotenuse

√(50² + 30²)

√(2500 + 900)

√3400

= 58.3 inches

y= -(x+3)^2 -5
What is the leading coefficient?
How do you find the vertex?

Answers

Answer:

To find the leading coefficient, first expand the function:

[tex]y= -(x+3)^{2} -5\\\\y=-(x^{2} +6x+9)-5\\\\y=-x^{2} -6x-9-5\\\\y=-x^{2} -6x-14[/tex]

The leading coefficient is the coefficient of the highest-order term, which, in this case, would be the -1 from -x².

To find the vertex: see image below

Vertex = (-3, -5)

3. If triangle ABC has the following measurements, find the measure of angle A.
a = 17
b = 21
C = 25

Answers

9514 1404 393

Answer:

  (a)  42.3°

Step-by-step explanation:

Side 'a' is the shortest of three unequal sides, so angle A will be the smallest angle in the triangle. Its measure can be found from the Law of Cosines.

  a² = b² +c² -2bc·cos(A)

  cos(A) = (b² +c² -a²)/(2bc) = (21² +25² -17²)/(2·21·25) = 777/1050

  A = arccos(777/1050) ≈ 42.3°

The measure of angle A is about 42.3°.

_____

Additional comment

The smallest angle in a triangle can never be greater than 60°. This lets you eliminate choices that exceed that value.

Answer:

 (a)  42.3°

Step-by-step explanation:

Which graph represents the function f(x)=|x−1|−3 ?

Answers

Answer is B, (the one on the top right)

Jo bought a used car for $6000 and paid a 15% deposit. How much did he still have to pay?

Answers

Answer:

900 is the correct awnser

John and Pablo caught fish that have the lengths, in centimeters, listed below. 45, 44, 47, 49, 45, 47, 42, 39, 45, 42, 44 Which box-and-whisker plot correctly represents the data?

Answers

The options for the box and whisker plots aren't given ; however using technology, a box and whisker plot could be generated from the data.

Answer:

Step-by-step explanation:

Given :

45, 44, 47, 49, 45, 47, 42, 39, 45, 42, 44

Using technology, the box and whisker plot generated for the data is attached below.

The 5 - number summary is also given below :

Minimum: 39

Median: 45

First quartile: 42

Third quartile: 47

Interquartile Range: 5

Maximum: 49

Outliers: none

Answer:

Step-by-step explanation:

What is an
equation of the line that passes through the points (-3,-1) and (-4,-4)

Answers

Answer:

y= 3x+8

Step-by-step explanation:

not a 100% sure...

sry if its wrong

(try using Math-way, its rly helpful)

Answer:

Step-by-step explanation:

y=mx+b

To find slope: -4+1/-4+3

Slope=3

y=3x+b

Plug in either points ,as an example, i'll plug in (-3,-1)

-1=3(-3)+b

-1=-9+b

8=b

Finished formula: y=3x+8

Graph the system of inequalities. Then state whether the situation is infeasible, has alternate optimal solutions, or is unbounded. (Assume that x>0 y>0

Answers

Answer:

x

[tex]x \leqslant - y + 1[/tex]

Steps are show in the picture above.

If YWZ=17, what is WXY?

34
56
17
73

Answers

Answer:

73

Step-by-step explanation:

17×2=34

180-34=146

146/2=73

=73

PLS HELP ME ON THIS QUESTION I WILL MARK YOU AS BRAINLIEST IF YOU KNOW THE ANSWER PLS GIVE ME A STEP BY STEP EXPLANATION!!
The students in Shawn's class got to choose whether to visit the zoo or the aquarium. 3 students went to the zoo and 15 students went to the aquarium. What is the ratio of the number of students who went to the zoo to the number of students who did not go to the zoo?
A. 1:6
B. 1:1
C. 1:3
D. 1:5

Answers

Answer:

either it's 1:3 or 1:6

Step-by-step explanation:

3 students went to the zoo, and 15 students went to the aquarium.

15 plus 3 is 18.

3/18 then reduces to 1/6

therefore it is:

1:6

(before, I got my answer deleted so, maybe it is 1:3, but I solved it and got 1:6)

HELP ASAP PLEASE!!!!!
what is the factorization of the polynomial below -x^2-2x+48

Answers

I HOPE IT WILL HELP YOU. I DID HOW I KNOW .

What is the difference between-5and2

Answers

Answer:

7

Step-by-step explanation:

Consider the absolute value of the difference , that is

| - 5 - 2 | = | - 7 | = 7

or

| 2 - (- 5) | = | 2 + 5 | = | 7 | = 7

Answer:

7

Step-by-step explanation:

Difference is - sign so the equation is: 2- -5 which is 7. Or

think a number line, -5 is 5 spots to 0, then two more spots to 2 so 5+2=7

Find m/ELM if m/ELM = 15x - 1, m/KLE = 20°, and m/KLM = 17x - 1.​

Answers

Answer:

∠ ELM = 149°

Step-by-step explanation:

∠ KLM = ∠ KLE + ∠ ELM  , substitute values

17x - 1 = 20 + 15x - 1

17x - 1 = 15x + 19 ( subtract 15x from both sides )

2x - 1 = 19 ( add 1 to both sides  )

2x = 20 ( divide both sides by 2 )

x = 10

Then

∠ ELM = 15x - 1 = 15(10) - 1 = 150 - 1 = 149°


What is the surface area of the right prism?
92 ft2
46 ft2
48 ft2
70 ft2

(will mark brainliest <3)

Answers

Answer:  70 ft^2   ( choice D )

=========================================================

Work Shown:

L = 8 ft = lengthW = 3 ft = widthH = 1 ft = height

SA = surface area of the rectangular prism (aka block or box)

SA = 2*(LW + LH + WH)

SA = 2*(8*3 + 8*1 + 3*1)

SA = 2*(24 + 8 + 3)

SA = 2*(35)

SA = 70 square feet

This is the amount of wrapping paper you would need to cover all six sides of the box. This assumes that there are no gaps or overlaps.

which statement is true

Answers

3) an $8 delivery fee and $1.50 per litre of water

This is because the $8 is a constant baseline, then adding $1.50 times the amount of litres purchased.

SOMEONE HELP ME PLEASE

Answers

Answer:

Step-by-step on:ơ

Select the two values of x that are roots of this equatio 2x - 5 = - 3x ^ 2

Answers

alright I can help!

so to find the two values of x that are roots of the equation we need to put the variables all on one side so that we can set up the quadratic formula.

3x^2+2x-5=0 (the -3x^2 becomes positive when moved across the equal sign)

now we can set up the quadratic formula. the equation is x= (-b+-(sqrt of b^2 -4ac))/ 2a

so now we just plug in our variables.

x= (-2+-(sqrt of 2^2 -4×3×-5))/ 2×3

x= (-2+-8)/6

now we just seperate the equations so that we have the two roots. and then just solve!

x= (-2-8)/6 -> x= -5/3

x= (-2+8)/6 -> x=1

hope this helps! best wishes and best of luck!!

What is the volume?
9 ft
4 ft
2 ft
HELPPPP

Answers

Answer:

72?

Step-by-step explanation:

V=whl=4 x 2 x9=72

Answer:

72 cubic feet

Step-by-step explanation:

*this formula doesn’t work for all shapes*

Volume = length x width x height

Volume = 4 x 2 x 9

Volume = 72 cubic feet

Pls help me ! L need help here

Answers

Answer:

H. 40 inches

Step-by-step explanation:

On Wednesday, he is 40 inches taller. ... That would make 5 days of growth, for 100 inches. But this is only 3 days therefore he would grow 40 inches taller

Guys please help me solve this I’m struggling

Answers

Answer:

[tex]Max\ z = 1[/tex]

[tex]Min\ z = -9[/tex]

Step-by-step explanation:

Given

[tex]z = 4x + 5y[/tex]

[tex]x \ge -1[/tex]

[tex]y \le 2x +3[/tex]

[tex]y \le -1[/tex]

Required

The maximum and minimum of z

To do this, we make use of the graphical method

See attachment for graphs of

[tex]x \ge -1[/tex]

[tex]y \le 2x +3[/tex]

[tex]y \le -1[/tex]

The corner points of the function are:

[tex](x,y) = (-1,1)[/tex]

[tex](x,y) = (-1,0)[/tex]

[tex](x,y) = (-1,-1)[/tex]

We have:

[tex]z = 4x + 5y[/tex]

Calculate z with  the above values

[tex]z = 4(-1) + 5(1) = 1[/tex]

[tex]z = 4(-1) + 5(0) = -4[/tex]

[tex]z = 4(-1) + 5(-1) = -9[/tex]

So, we have:

[tex]Max\ z = 1[/tex]

[tex]Min\ z = -9[/tex]

Other Questions
PLEASE HELP ASAPSolve the inequality [tex]\sqrt[3]{x+4} \ \textgreater \ \sqrt[2]{-x}[/tex]A) x < 2B) x > 2C) x > 2D) x < 2 The assertion that non formal institution have no role to play in the processing stage of the system theory is an empty rhetoric Standish Company manufactures consumer products and provided the following information for the month of February: Units produced 131,000 Standard direct labor hours per unit 0.20 Standard fixed overhead rate (per direct labor hour) $2.50 Budgeted fixed overhead $65,000 Actual fixed overhead costs $68,300 Actual hours worked 26,350Required:a. Calculate the fixed overhead spending variance using the formula approach.b. Calculate the volume variance using the formula approach. [tex]\sqrt{x - 5} - \sqrt{3 - 2.x} = 0[/tex] Which of these sentences correctly uses the comparative form of an adverb?A. The director gives better presentations than other team members.B. He gave the best presentation.C. The most best dancer will be given an award at the party.D. She paints so well it is almost better than the art teacher. An increase in the wage rate causes Group of answer choices a decrease in the quantity of labor demanded. an increase in labor's marginal productivity. a leftward shift of the firm's labor demand curve. a rightward shift of the firm's labor demand curve. upon receiving your first salary, you deposited 3000 taka monthly in a fund for your future for 18 years. the fund earns 6% interest rate compounded monthly. after 18 years, you want it to make payments at the end of every quarter for five year 4.5% compounded quarterly, what is the amount of each annuity payment to you? Which is a perfect square?61626365 create a graph of 4.95 + 3.99 QUESTION 18Indicate the correct statement:(1) An ex parte application is the motion application used when there is no dispute of facts(2) Action proceedings is another type of motion application and the procedure is started by anotice of motion(3) In criminal cases a procedure is started with action proceedings(4) The parties in a criminal case are the prosecutor and the defendant All of the following statements are true regarding non-government organizations EXCEPT: Una especie de mariposa presenta rasgos fenotpicos diferentes entre los miembros de sus poblaciones. Estos rasgos van desde una coloracin verdosa y antenas cortas (caractersticas recesivas). El fenotipo ms comn (caractersticas dominantes) de esta especie es de coloracin amarilla y de antenas tan largas que se pueden doblar. Realice el cruce entre dos mariposas que presentan fenotipo ms comn, pero de genotipos heterocigotos y determine los posibles rasgos fenotpicos que se pueden presentar. Let h(x)=20e^kx where k R (Picture attached. Thank you so much!) What electrical appliances do people in your country have at home? A binary system of species 1 and 2 consists of vapor and liquid phases in equilibriumat temperature T. The overall mole fraction of species 1 in the system is z1 = 0.65. Attemperature T, ln1 = 0.67 x22; ln2 = 0.67 x12; P1sat = 32.27 kPa; and P2sat = 73.14 kPa.Assuming the validity of Eq. (13.19),Final PDF to printer13.10. Problems 511smi96529_ch13_450-523.indd 511 01/06/17 03:27 PM(a) Over what range of pressures can this system exist as two phases at the given T and z1?(b) For a liquid-phase mole fraction x1 = 0.75, what is the pressure P and what molarfraction of the system is vapor? (c) Show whether or not the system exhibits an azeotrope Fill in the blanks.(3b^3)^2 = _b^_ In the short run, open-market purchases a. increase investment and real GDP, and decrease interest rates. b. increase real GDP and interest rates, and decrease investment. c. increase investment and interest rates, and decrease real GDP. d. decrease investment, interest rates, and real GDP. hi, please solve these three questions for me, i have to shoe solving steps. The faculty at Sherwood Elementary School has met to discuss peer-tutoring ideas to incorporate into their classrooms due to students' poor performance on the recent standardized achievement scores. One teacher suggests that a committee of teachers first identify students' areas of need and then target other students excelling in those areas that would be available to serve as tutors. Which peer tutoring program does this sound most like Five consecutive multiples of 7 have a sum of 350. What is the smallest of these numbers?A. 70B. 56C. 77D. 84